1answer.
Ask question
Login Signup
Ask question
All categories
  • English
  • Mathematics
  • Social Studies
  • Business
  • History
  • Health
  • Geography
  • Biology
  • Physics
  • Chemistry
  • Computers and Technology
  • Arts
  • World Languages
  • Spanish
  • French
  • German
  • Advanced Placement (AP)
  • SAT
  • Medicine
  • Law
  • Engineering
vovikov84 [41]
2 years ago
8

A loop rests in the plane of a page of textbook while a magnetic field is directed into the page. A clockwise current is induced

.
Physics
1 answer:
weqwewe [10]2 years ago
7 0

(b) when the magnetic field gets stronger.

correct option :

When a loop rest in the plane of the page and magnetic field is directed into the page then we will apply here right hand thumb rule in which thumb represent the direction of magnetic field and curling fingers represent direction of induced current. So, in this case the direction of magnetic field is into the page and curling fingers shows induced current in clockwise direction.

  Thus, option (b) is correct.

Incorrect options:

   option (A) is incorrect because when magnetic field will change  

     according to flux.

   option (C) is incorrect because field is stronger so it does not matter

     size of loop increases or decreases.

   option (D) is incorrect because it moves any other side the field will be

     stronger.

Learn more about magnetic field induction here:

 brainly.com/question/13610297

   #SPJ4

[Your question is incomplete, but most probably your full question was-Check all that apply:

A. when the magnetic field is tilted so it is no longer perpendicular to the page

B. when the magnetic field gets stronger

C. when the size of the loop decreases

D. when the loop is moved sideways across the page]

You might be interested in
Just after a motorcycle rides off the end of a ramp and launches into the air, its engine is turning counterclockwise at 8325 re
alexgriva [62]

Answer:

\frac{Ie}{lm} = 1.10*10^{-3}

Explanation:

GIVEN DATA:

Engine operating speed nf = 8325 rev/min

engine angular speed ni= 12125 rev/min

motorcycle angular speed N_m= - 4.2 rev/min

ratio of moment of inertia of engine to motorcycle is given as

\frac{Ie}{lm} = \frac{-N}{(nf-ni)}

\frac{Ie}{lm} = \frac{-(-4.2)}{(12125 - (8325))}

\frac{Ie}{lm} = 1.10*10^{-3}

5 0
3 years ago
Read 2 more answers
The product of 14 and a cubed
olya-2409 [2.1K]

Answer:

14 × a^3

Explanation:

Product means multiplication

Cubed means to the power of 3

6 0
3 years ago
Read 2 more answers
PLS HELP WILL MARK BRAINLIEST IF RIGHT NEED IMMEDIATELY PLEASEEEEE
timofeeve [1]

Answer:

Explanation:

Prescribe medications.

Prescribe treatments or therapies.

Treat patients using psychological therapies.

Collect medical information from patients, family members, or other medical professionals.

Record patient medical histories.

Develop medical treatment plans

3 0
2 years ago
Read 2 more answers
What force does a trampoline have to apply to a gymnast to accelerate her straight up at ? Note that the answer is independent o
Andrew [12]

Answer: Force applied by trampoline = 778.5 N

<em>Note: The question is incomplete.</em>

<em>The complete question is : What force does a trampoline have to apply to a 45.0 kg gymnast to accelerate her straight up at 7.50 m/s^2? note that the answer is independent of the velocity of the gymnast. She can be moving either up or down or be stationary. </em>

Explanation:

The total required the trampoline by the trampoline = net force accelerating the gymnast upwards + force of gravity on her.

= (m * a) + (m * g)

= m ( a + g)

= 45 kg ( 7.50 *  9.80) m/s²

Force applied by trampoline = 778.5 N

5 0
2 years ago
Dos cargas puntuales están fijas en el eje x: q1 = 6.0µC está en el origen, O, con x1 = 0.0 cm, y q2 = –3.0 µC está situada en e
erik [133]

Answer:

E_total = 1.30 10¹⁰ C / m²

Explanation:

The intensity of the electric field is

     E = k q / r²

on a positive charge proof

The total electric field at the midpoint is

as q₁= 6 10⁻⁶ C the field is outgoing to the right

for charge q₂ = -3 10⁻⁶ C, the field is directed to the right, therefore

E_total = E₁ + E₂

E_total = k q₁ / r₁² + k q₂ / r₂²

r₁ = r₂ = r = 4 10⁻² m

E_total = k/r² (q₁ + q₂)

 we calculate

E_total = 9 10⁹ / (4 10⁻²)²   (6.0 10⁻⁶ +3.0 10⁻⁶)

E_total = 1.30 10¹⁰ C / m²

8 0
3 years ago
Other questions:
  • You are fixing the roof of your house when a hammer breaks loose and slides down. The roof makes an angle of 65o∘ with the horiz
    6·1 answer
  • how does electromagnetism effect the function of a generator (using magnetism to produce electrical energy)
    12·1 answer
  • 1) There are many positive and negative aspects associated with nuclear power. Which is a negative aspect associated with nuclea
    14·1 answer
  • When a cup is placed on a table, which force prevents the cup from falling to the ground? A. gravitational force B. normal force
    8·2 answers
  • The cars salemens tells you that the car can go from a stopped position to 60 miles per hour in 6 seconds he is giving you the c
    14·1 answer
  • A ball is projected vertically downward at a speed of 4.00 m/s. How far does the bal travel in 1.80 s? What is the velocity of t
    10·1 answer
  • What is the fastest thing in the world, and what is its speed?
    8·1 answer
  • 7. A ball of mass m makes a head-on elastic collision with a second ball (at rest) and rebounds with a speed equal to 0.450 its
    6·1 answer
  • PLEASE TRY TO ANSWER AS MANY QUESTIONS AS YOU CAN !
    12·1 answer
  • How many types of ion are formed when water splits during
    5·1 answer
Add answer
Login
Not registered? Fast signup
Signup
Login Signup
Ask question!